Mathcenter Forum  

Go Back   Mathcenter Forum > คณิตศาสตร์มัธยมศึกษา > ปัญหาคณิตศาสตร์ ม. ต้น
สมัครสมาชิก คู่มือการใช้ รายชื่อสมาชิก ปฏิทิน ข้อความวันนี้

ตั้งหัวข้อใหม่ Reply
 
เครื่องมือของหัวข้อ ค้นหาในหัวข้อนี้
  #1  
Old 31 กรกฎาคม 2011, 17:29
Worrchet Worrchet ไม่อยู่ในระบบ
บัณฑิตฟ้า
 
วันที่สมัครสมาชิก: 15 พฤษภาคม 2011
ข้อความ: 373
Worrchet is on a distinguished road
Default เลขคณิตและพีชคณิต2 ขอโทษทีนะครับอันเก่ามันไม่แสดงผลเป็นLatex

ขอถามหน่อยนะครับ ใครเก่งวิทย์(คำนวณ ชีวะ ฟิสิก ...)และก็ภาษาอังกฤษบ้างครับ ผมมีเรื่องจะให้ช่วยหน่อยขอบคุณครับ
มีมาเพิ่ม update (5/08/54)

1.ถ้า $\frac{1}{8}=\frac{1}{2+2\sqrt[]{a+\sqrt[]{a+\sqrt[]{a+...}}}}$ และ a มีค่าเท่าใด(โอลิมปิกรอบแรก)
กำหนด $x=\frac{1}{\sqrt[]{a+\sqrt[]{a+\sqrt[]{a+...}}}}$
จากโจทย์ ; $\frac{1}{8}=\frac{1}{2+2x}$<---จากที่โจทย์กำหนดมาว่่า $x=\frac{1}{\sqrt[]{a+\sqrt[]{a+\sqrt[]{a+...}}}}$ เวลาแทนค่าใน $\frac{1}{2+2x}$ ค่ามันไม่กลายเป็น$\frac{1}{2+2(\frac{1}{\sqrt[]{a+\sqrt[]{a+\sqrt[]{a+...}}}})}$หรอครับ
            $x=3$
ดังนั้น $a+\sqrt[]{a+\sqrt[]{a+\sqrt[]{a+...}}}=9$<---มาจากไหน
$a+x=9$
$a=6$

2.$24^n$ หาร $100!$ ลงตัว จงหาค่า $n$ ที่มากที่สุดที่เป็นไปได้(มหิดล)
เนื่องจาก $\frac{100!}{24^n}$ =$\frac{100!}{(2^{3n})(3^n)}$
$100!$ หารด้วย $(2^{3n})(3^n)$ ลงตัวก็ต่อเมื่อ เราทราบว่า $100!$มีตัวประกอบ $2,3$ อยู่กี่ตัวโดยอาศัยสูตรของเลอจองดร์ดังนี้<---อยากทราบว่าทำไมต้องใช้สูตรของเลอจองดร์?
พิจารณา $100!$ จะมี 2 เป็นตัวประกอบ
$=[\frac{100}{2}][\frac{100}{2^2}][\frac{100}{2^3}][\frac{100}{2^4}][\frac{100}{2^5}][\frac{100}{2^6}]$
$=50+25+12+6+3+1$
$=97$ตัว
$100!$จะมี3เป็นตัวประกอบกี่ตัว
$=[\frac{100}{3}][\frac{100}{3^2}][\frac{100}{3^3}][\frac{100}{3^4}]$
$=33+11+1+3+1$
$=48$ตัว
ดังนั้น $\frac{100!}{24^n}=\frac{2^{97}\cdot3^{48}\cdot...\cdot97}{(2^{3n})(3^{n})}$<---คูณไปเรื่อยๆจนถึง97หมายความว่าอย่างไร
(ช่วยอธิบายข้อความข้างล่างนี้หน่อยครับ)
ทำให้เราสรุปได้ว่า $3n<97$ และ $n<48$จึงทำให้มีผลลัพธ์ของ$\frac{100!}{24^n}$ลงตัวจะได้ $n=32$

3.จงหาจน.เต็มบวก N มีน้อยที่สุดซึ่งเมื่อหารด้วย 10,9,7,6,5,4,3,2จะเหลือเศษ9,8,7,6,5,4,3,2,1ตามลำดับแต่เมื่อหารด้วย 11 จะเหลือเศษ 2 (โอลิมปิก)<---จากโจทย์ผมงงว่าจน.ตัวหารไม่เท่ากับจน.เศษคือ ตัวหารมี 8 ตัว ตัวเศษมี 9ตัว?

ครน.ของ 10,9,7,6,5,4,3,2 เท่ากับ $m=2520$
จากโจทย์จะได้ 10,9,7,6,5,4,3,2 หาร $n+1$ ลงตัว
ฉะนั้น  $n+1$  เป็นตัวคูนร่วมของ 10,9,7,6,5,4,3,2 
ทำให้ได้ว่า $m|(N+1)$ นั่นคือ $2520|(N+1)$
;$(n+1)=2520,5040,7560,...$<---มาจากไหนครับหรือว่าเกิดจาก  2520+ 2520=5040ใช่ไหมครับถ้าถูกช่วยอธิบายด้วยว่าบวกกันทำไม
ดังนั้นN=2519,5039,7559,...
จากที่ Nหารด้วย 11 เหลือเศษ2
จะได้ $N=7559$ เป็นจน.เต็มที่น้อยที่สุด<---ช่วยขยายความหน่อยและทำไม $N=7559$

4.ถ้า $2^{\frac{1}{2}}\cdot4^{\frac{1}{4}}\cdot8^{\frac{1}{8}}\cdot...\cdot1024^{\frac{1}{1024}}$ สามารถเขียนอยู่ในรูป $2^{2-\frac{a}{b}}$ ค่าของ $a-b+1$ มีค่าเท่าใด(มหิดล)

$2^{\frac{1}{2}}\cdot4^{\frac{1}{4}}\cdot8^{\frac{1}{8}}\cdot...\cdot1024^{\frac{1}{1024}}$
=$2^{\frac{1}{2}}\cdot(2^2)^{\frac{1}{4}}\cdot(2^3)^{\frac{1}{8}}\cdot...\cdot(2^{10})^{\frac{1}{1024}}$
$=2^{\frac{1}{2}+\frac{2}{2^2}+\frac{3}{2^3}+...+\frac{10}{2^{10}}}$     -----(1)
พิจารณา $s={\frac{1}{2}+\frac{2}{2^2}+\frac{3}{2^3}+...+\frac{10}{2^{10}}}$     -----(2)
$(1)\cdot2;2s=1+\frac{2}{2}+\frac{3}{2^2}+...+\frac{10}{2^9}$     -----(3)
(ช่วยอธิบายข้อความข้างล่างนี้หน่อยนะครับงง)
$(3)-(2);s=1+(\frac{1}{2}+\frac{1}{2^3}+...+\frac{1}{9^9})-\frac{10}{2^{10}}$
s=$1+\frac{\frac{1}{2}(1-\binom{1}{2}^9)}{1-\frac{1}{2}}-\frac{10}{2^{10}}$
s=$1+(1-\frac{1}{2^9})$-$\frac{10}{2^{10}}$
s=$\frac{2^{11}-2-10}{2^{10}}$
s=$\frac{2048-12}{1024}$
s=$\frac{509}{256}$

5.กำหนดให้ $P(x)$ เป็นพหุนามที่มีเศษเหลือเท่ากับ 3 เมื่อหารด้วย $x-2$และมีเศษเหลือเท่ากับ 5 เมื่อหารด้วย $x-3$ เศษเหลือที่ได้จาก P(x) หารด้วย $x^{2}-5x+6$ เท่ากับเท่าใด(สมาคมคณิตศาสตร์)

กำหนด $P(x)=Q(x)(x^{2}-5x+6)+(ax+b)$<---นี่คือรูปแบบของทฤษฎีเศษเหลือใช่ไหมครับแล้วแต่ละตัวมีหน้าที่ดังนี้ 
$P(x)$=ตัวตั้ง
$Q(x)$=ตัวหาร
$(x^{2}-5x+6)$=ผลหาร
$(ax+b)$=เศษ  
ถูกต้องไหมครับ.
โจทย์กำหนด $P(x)/(x-2)$เหลือเศษ 3 
$;2a+b=3$     -----(1)<---เกิดจากการแทนค่า $x=2$ ใช่ไหมครับ ผมงงว่่า เราสามารถดึงค่ามันออกมาได้ด้วยหรอครับช่วยอธิบายหน่อยครับแล้วพอแทนค่าลงไปแล้ว$P$ หายไปไหน.
และจากโจทย์กำหนด $P(x)/(x-3)$เหลือเศษ 3 
$;3a+b=5$     -----(2)
(2)-(1) ;$a=2,b=-1$
ดังนั้น เศษเหลือที่ได้จาก $P(x)$หารด้วย $(x^{2}-5x+6)$ เท่ากับ $2x-1$($2x-1$มาจากไหนครับ)

6.จน.เต็มบวก n ที่มีค่าน้อยที่สุดซึ่งทำให้ $(2000n+1)(2008n+1)$ เป็นกำลังสองสมบูรณ์ที่มีค่าเป็นเท่าใด(มหิดล)
กำหนด $(2000n+1)(2008n+1)$ เป็นจน.กำลังสองสมบูรณ์
แสดงว่า จะมีจน.เต็มบวก m ที่ทำฝให้
$(2000n+1)(2008n+1)=m^{2}$
$(2000•2008)n^{2}+4008n=m^{2}-1$
$n[(250•2008)n+501]=(m+1)(m-1)$
เนื่องจาก$n=1,2,3,...,500$ไม่สามารถนำไปหาค่า m ได้<---หมายความว่าอย่างไรครับ.
แทนค่า $n=501$ $;8n[(250•2008)n+501]$<---แล้วรู้ได้อย่างไรว่า $n=501$มันเป็นแค่การนำ 8 ไปหารไม่ใช่หรอครับ?.
$=4008[(250•2008•501)+501]$
$=1004004•1004002$
แสดงว่า n ที่น้อยที่สุดที่ทำให้หา m ได้คือ 501
ดังนั้น ค่า n ที่น้อยที่สุดที่ทำให้ $(2000n+1)(2008n+1)$เป็นจน.กำลังสองสมบูรณ์ คือ 501 <---$n=501$เกี่ยวอะไรกับกำลังสองสมบูรณ์ครับ

7.กำหนด a,b,c,d,e เป็นจน.เต็มซึ่งแตกต่างกันทั้งหมดและสอดคล้องกับสมการ (2009-a)(2009-b)(2009-c)(2009-d)(2009-e)=325 แล้ว a+b+c+d+e มีค่าเท่าใด(มหิดล)
ข้อนี้ต้องพยายามแยกตัวประกอบของ  325 ให้อยู่ในรูปการคูณของจน. 5 จน.
เนื่องจาก 325 = 25•13
                  =5•5•13
                  =(-5)•5•(-1)•1•13<---ทำไมต้องคูณ (-5),(-1)และทำไมไม่คูณ (-13)เข้ามาด้วยครับ
อธิบายตั้งแต่ค่า a ถึง e ว่าออกมาเป็นแบบนี้ได้อย่างไรและนำมาลบทำไม
;$a=2009+5$
 $b=2009-5$
 $c=2009+1$
 $d=2009-1$
 $e=2009-1$<---ทำไม(-1)เหมือนกัน
ดังนั้น a+b+c+d+e=10032

8.ใหั m และ n เป็นจน.เต็มบวกที่สอดคล้องกับสมการ $\frac{1}{m}+\frac{1}{n}=\frac{19}{94}$ จงหาค่า m+n (มหิดล)

กำหนด $\frac{1}{m}+\frac{1}{n}=\frac{19}{94}$
$\frac{1}{m}=\frac{19}{94}-\frac{1}{n}$
$\frac{1}{m}=\frac{19n-94}{94n}$
ดังนั้น  $m=\frac{94n}{19n-94}$
แสดงว่า $19n-94$ หาร $94n$ ลงตัว<---รู้ได้อย่างไร
ดังนั้น $19n-94=1$ <---ทำไมต้องเท่ากับ 1.
n=5
จาก $m=\frac{94n}{19n-94}=\frac{94(5)}{19(5)-94}=470$

9.จน.เต็มบวก 4 จน.ต่างๆกัน คือ $a,b,c,N$ โดยที่ $N=5a+3b+5c$ และ $N=4a+5b+4c$ โดยที่ $131<N<150$ จงหาค่าของ $a+b+c$ (มหิดล)

$N=5a+3b+5c=4a+5b+4c$
$a-2b+c=0$
$a+c=0$     -----(1)<---(-2b)หายไปไหนครับ
ดังนั้น $a+b+c =3b$ <---(3b)มาจากไหนคร้บ
$N=5a+3b+5c$
$N=5(a+c)+3b$
$N=5(2b)+3b$
$N=13b$
เนื่องจาก $N,b$ เป็นจน.เต็มบวก
โดยที่ $131<N<150$
N จะต้องเป็นพหูคูณของ 13 (เนื่องจาก N=13b)<---ช่วยขยายความและยกตัวอย่างหน่อยครับ
ซึ่ง N ที่เป็นไปได้ คือ $N=143$ และ $b=11$
แทนค่า $b=11$ ใน (1) จะได้
$a+c=22$
ดังนั้น $a+b+c=33$
ดังนั้น m+n=475

10.ถ้า a และ b เป็นจน.จริงที่ $(60)^{a}=3$ และ $(60)^{b}=5$
แล้วค่าของ $(12)^{\frac{1-a-b}{2(1-b)}}$ เท่ากับ (โอลิมปิก)

เนื่องจาก 
$12=\frac{60}{5}$
$12=\frac{60}{60^{b}}$
$12=60^{1-b}$
;$(12)^{\frac{1-a-b}{2(1-b)}}$ <---มาจากไหนครับ
$=(60^{1-b})^{\frac{1-a-b}{2(1-b)}}$<---มันแทนค่าอย่างไรครับและ มาจากไหนครับ. 
$(12)^{\frac{1-a-b}{2(1-b)}}$ $=60^{\frac{1-a-b}{2}}$
$=(\frac{60}{60^{a}\cdot60^{b}})^{\frac{1}{2}}$ <---ออกมาเป็นแบบนี้อย่างไรครับ.
.
.
.

11.กำหนด $x$ และ $y$ เป็นจน.เต็มบวก สมการ $\frac{1}{x}+\frac{1}{y}=\frac{1}{2008}$ มีทั้งหมดกี่จน. (สพฐ.รอบที่2)

กำหนด $\frac{1}{x}+\frac{1}{y}=\frac{1}{2008}$
$2008x+2008y=xy$
$xy-2008x-2008y+2008^{2}=2008^{2}$
$(x-2008)(y-2008)=2008^{2}$ <---มาอยู่ในรูปนี้ได้อย่างไร.
$(x-2008)(y-2008)=(2^{3}\cdot251)^{2}$
$(x-2008)(y-2008)=2^{6}•251^{2}$
(อธิบายตั้งแต่ข้างล่างนี้ลงไปแบบละเอียดเลยนะครับ)
แสดงว่า คำตอบของสมการคือ จน.เต็มบวก x ซึ่ง
$x=2008+2^{a}•251^{b}$ โดยที่ $a=0,1,2,3,4,5,6$
$b=0,1,2,$
หรือ $x=2008-2^{a}•251^{b}$
 โดยที่ $a=0,1,2,3$
$b=0,1$
เนื่องจาก $x \neq 2008-1$
 และ $x\neq$ 2008-2008
ดังนั้น จน.คำตอบของสมการ =$(7\cdot3)$+$(4\cdot2)$-2=27

05 สิงหาคม 2011 12:35 : ข้อความนี้ถูกแก้ไขแล้ว 5 ครั้ง, ครั้งล่าสุดโดยคุณ Worrchet
เหตุผล: เพิ่มโจทย์
ตอบพร้อมอ้างอิงข้อความนี้
  #2  
Old 31 กรกฎาคม 2011, 22:14
poper's Avatar
poper poper ไม่อยู่ในระบบ
กระบี่ธรรมชาติ
 
วันที่สมัครสมาชิก: 12 พฤษภาคม 2010
ข้อความ: 2,643
poper is on a distinguished road
Send a message via MSN to poper
Default

ข้อ 3 อธิบายอย่างละเอียดเลยแบบนี้นะครับ
เลขยกกำลังทั้งหมดคือ $$(2!+3!+4!+...+2547!)^{(3!+4!+5!+...+2547!)^{...(2547!)}}$$
เนื่องจาก $(2!+3!+4!+...+2547!)$ หารด้วย 4 ลงตัว ดังนั้นไม่ว่าจะยกกำลังอะไรก็ยังคงหารด้วย 4 ลงตัวเหมือนเดิม
ดังนั้นเลขยกกำลังข้างบนจึงไม่ต้องไปคิด แล้วกำหนดให้เป็น $h$ไป จึงได้ เลขยกกำลังคือ $$(2!+3!+4!+...+2547!)^h$$
$$x=(1!+2!+3!+4!+...+2547!)^{(2!+3!+4!+...+2547!)^h}$$
เรารู้แล้วว่า เลขยกกำลังนั้น หารด้วย 4 ลงตัว จึงกำหนดให้ $$(2!+3!+4!+...+2547!)^h=4k$$
(เพราะจำนวนที่หารด้วย 4 ลงตัวย่อมมี4เป็นตัวประกอบแน่นอน เช่น 12=4x3,48=4x12 เป็นต้น)
จึงได้ว่า $$x=(1!+2!+3!+4!+...+2547!)^{4k}$$
และรู้แล้วว่า $1!+2!+3!+4!+...+2547!$ ลงท้ายด้วย $3$ ดังนั้น
$x=(.....3)^{4k}=\{(....3)^4\}^k=(....1)^k=....1$
__________________
คณิตศาสตร์ คือ ภาษาสากล
คณิตศาสตร์ คือ ความสวยงาม
คณิตศาสตร์ คือ ความจริง
ติดตามชมคลิปวีดีโอได้ที่http://www.youtube.com/user/poperKM
ตอบพร้อมอ้างอิงข้อความนี้
  #3  
Old 31 กรกฎาคม 2011, 22:22
Worrchet Worrchet ไม่อยู่ในระบบ
บัณฑิตฟ้า
 
วันที่สมัครสมาชิก: 15 พฤษภาคม 2011
ข้อความ: 373
Worrchet is on a distinguished road
Default

ผมงงว่าทำไมมันมารวมกันอย่างนั้นอะครับช่วยด้วยทำไม่เป็น
ตอบพร้อมอ้างอิงข้อความนี้
  #4  
Old 31 กรกฎาคม 2011, 22:48
poper's Avatar
poper poper ไม่อยู่ในระบบ
กระบี่ธรรมชาติ
 
วันที่สมัครสมาชิก: 12 พฤษภาคม 2010
ข้อความ: 2,643
poper is on a distinguished road
Send a message via MSN to poper
Default

หมายถึงเลขยกกำลังใช่มั้ยครับ ลองนึกแบบนี้ครับ
$4$ หารด้วย $4$ ลงตัว
$4^2=4\cdot4$ หารด้วย $4$ ลงตัว
$4^{2^3}=4^8$ หารด้วย $4$ ลงตัว
$4^{2^{3^2}}=4^{512}$ หารด้วย $4$ ลงตัว
.
.
.
ดังนั้นไม่ว่าจะยกกำลังเท่าไหร่ก็หารด้วย 4 ลงตัวเสมอครับ
ในกรณีที่เลขยกกำลังซ้อนกันเยอะๆ เราจะไม่ไปนั่งคิดหรอกนะครับ ก็จะกำหนดไปเป็นตัวแปรเอาครับ
เช่น$$4^{2^{3^{4^{5^6}}}}=4^k$$ หารด้วย 4 ลงตัว
__________________
คณิตศาสตร์ คือ ภาษาสากล
คณิตศาสตร์ คือ ความสวยงาม
คณิตศาสตร์ คือ ความจริง
ติดตามชมคลิปวีดีโอได้ที่http://www.youtube.com/user/poperKM
ตอบพร้อมอ้างอิงข้อความนี้
  #5  
Old 02 สิงหาคม 2011, 00:09
Mol3ilE Mol3ilE ไม่อยู่ในระบบ
ลมปราณบริสุทธิ์
 
วันที่สมัครสมาชิก: 29 กรกฎาคม 2011
ข้อความ: 130
Mol3ilE is on a distinguished road
Default

รบกวนคุณ poperอธิบายข้ออื่นด้วยครับขอบคุณครับ

ข้อ8อ่ะครับ โจทย์ผิดหรืิอป่าวมันบวก44444....4444---100ตัวไม่ใช่หรอครับ ถ้า1000มันจะได้คำตอบอื่นนิ

02 สิงหาคม 2011 00:36 : ข้อความนี้ถูกแก้ไขแล้ว 1 ครั้ง, ครั้งล่าสุดโดยคุณ nongtum
เหตุผล: double post+แก้เล็กน้อยโปรดใช้ปุ่มแก้ไข
ตอบพร้อมอ้างอิงข้อความนี้
  #6  
Old 02 สิงหาคม 2011, 00:55
poper's Avatar
poper poper ไม่อยู่ในระบบ
กระบี่ธรรมชาติ
 
วันที่สมัครสมาชิก: 12 พฤษภาคม 2010
ข้อความ: 2,643
poper is on a distinguished road
Send a message via MSN to poper
Default

ข้อ 4
ที่บอกว่า $m!-n!\not=(m-n)!$ เพราะเห็นคุณ Worrchet อธิบายมาแบบนั้นครับ
$(2!-1!)+(3!-2!)+(4!-3!)+(5!-4!)+...+(19!-18!)+(20!-19!)+(21!-20!)$
จะเห็นว่าตัวหน้าของแต่ละวงเล็บจะไปตัดกับตัวหลังของวงเล็บถัดไปเสมอ
ดังนั้นตัวที่เหลืออยู่คือ $-1!$ ของวงเล็บแรก และ $21!$ ของวงเล็บสุดท้ายครับ
ข้อ 5
จากบรรทัด $4+(\frac{1}{2}+\frac{1}{6}+\frac{1}{12}+\frac{1}{20})$ ทำให้เราทราบว่า
อนุกรมมีรูปแบบ คือ $$n+\sum_{k=1}^{n}(\frac{1}{k(k+1)})$$(ลองแทน $n=4$ จะได้ตามบรรทัดบนครับ)
ดังนั้น จากโจทย์เป็นการบวกกัน 1999 พจน์ นั่นคือ $n=1999$ จะได้
$1999+(\frac{1}{2}+\frac{1}{6}+\frac{1}{12}+\frac{1}{20}+...+\frac{1}{1999\cdot2000})$ ครับ
ข้อ 9 น่าจะพิมพ์ผิดครับ ที่ถูกควรจะเป็น
$\frac{1^4+n^4+(n+1)^4}{1^2+n^2+(n+1)^2}$ ครับ ถามว่าสมมุติมาจากไหน ก็ลองแทน $n=2$ จะได้ตัวแรก $n=3$ ก็จะได้ตัวที่สอง
แบบนี้ไปเรื่อยๆครับ
ข้อ 10
อยู่ที่การจัดกลุ่มครับ แต่เราต้องรู้ว่า ถ้ามี $a+b$ ต้องคูณด้วย $a-b$ ถ้ามี $a-b$ก็ต้องคูณด้วย $a+b$ ครับ
จากข้อนี้คือ $1+\sqrt{2}-\sqrt{3}$ เราจัดกลุ่มต่างๆได้ดังน้
$1+(\sqrt{2}-\sqrt{3})$ กรณีนี้ต้องคูณด้วย $1-(\sqrt{2}-\sqrt{3})$
$(1+\sqrt{2})-\sqrt{3}$ กรณีนี้ต้องคูณด้วย $(1+\sqrt{2})+\sqrt{3}$
$(1-\sqrt{3})+\sqrt{2}$ กรณีนี้ต้องคูณด้วย $(1-\sqrt{3})-\sqrt{2}$
ข้อ 11
พิมพ์ผิดนะครับต้องเป็น $\frac{\sqrt{93-2\sqrt{75\cdot18}}}{3\sqrt{6}}$
แล้วข้างบนจะเข้าสูตรนี้ครับ $\sqrt{a}-\sqrt{b}=\sqrt{(a+b)-2\sqrt{ab}}\ \ ,a>b$
ข้อ 12 $\sqrt{3+\sqrt{5}}\cdot\sqrt{3-\sqrt{5}}=\sqrt{(3+\sqrt{5})(3+\sqrt{5})}=\sqrt{4}=2$ ครับ สงสัยหนังสือจะผิด
ข้อ 13
$(9+\sqrt{77})^{\frac{1}{2}}=A\ \ ,(9-\sqrt{77})^{\frac{1}{2}}=B$
$A=\sqrt{9+\sqrt{77}}=\frac{\sqrt{18+2\sqrt{77}}}{\sqrt{2}}=\frac{\sqrt{11}+\sqrt{7}}{\sqrt{2}}$
ในทำนองเดียวกัน $B=\frac{\sqrt{11}-\sqrt{7}}{\sqrt{2}}$
จากโจทย์ แทนค่าจะได้
$\frac{A^3-B^3}{10\sqrt{14}}=\frac{(A-B)(A^2+AB+B^2)}{10\sqrt{14}}$
$=\frac{(\sqrt{14})(22)}{10\sqrt{14}}=\frac{11}{5}$
ข้อ 14
ลองให้ $A=a^2+9a$ ครับจะได้
$A(A+18)=A^2+18A$
แทนค่ากลับจะได้
$(a^2+9a)^2+18(a^2+9a)$
ข้อ 15
สังเกตุว่าในรูทมี เลข 1 สิบสองหลัก คูณกับ เลขสิบสองหลักเช่นเดียวกันแล้วบวกด้วย 1
1. นำ 9 คูณและหาร ในรูท จะได้
$\sqrt{\frac{9[(111111111111)(1000000000005)+1]}{9}}=\frac{\sqrt{(99999999999)(1000000000000+5)+9}}{3}$
$=\frac{1}{3}\sqrt{(99999999999)(1000000000000+5)+9}$ ($999999999999=10^{12}-1$)
$=\frac{1}{3}\sqrt{(10^{12}-1)(10^{12}+5)+9}$ แล้วคูณกระจาย จะได้
$\frac{1}{3}\sqrt{(10^{24}+4\cdot10^{12}-5)+9}$
$=\frac{1}{3}\sqrt{(10^{12})^2+4\cdot10^{12}+4}$ เข้าสูตร $a^2+4a+4=(a+2)^2$
$=\frac{1}{3}\sqrt{(10^{12}+2)^2}$
__________________
คณิตศาสตร์ คือ ภาษาสากล
คณิตศาสตร์ คือ ความสวยงาม
คณิตศาสตร์ คือ ความจริง
ติดตามชมคลิปวีดีโอได้ที่http://www.youtube.com/user/poperKM
ตอบพร้อมอ้างอิงข้อความนี้
  #7  
Old 02 สิงหาคม 2011, 01:20
poper's Avatar
poper poper ไม่อยู่ในระบบ
กระบี่ธรรมชาติ
 
วันที่สมัครสมาชิก: 12 พฤษภาคม 2010
ข้อความ: 2,643
poper is on a distinguished road
Send a message via MSN to poper
Default

อ้างอิง:
ข้อความเดิมเขียนโดยคุณ Mol3ilE View Post
รบกวนคุณ poperอธิบายข้ออื่นด้วยครับขอบคุณครับ

ข้อ8อ่ะครับ โจทย์ผิดหรืิอป่าวมันบวก44444....4444---100ตัวไม่ใช่หรอครับ ถ้า1000มันจะได้คำตอบอื่นนิ
จริงด้วยครับ ถ้าจะให้ตรงกับที่เฉลยต้องเป็น 100 ตัวครับ
__________________
คณิตศาสตร์ คือ ภาษาสากล
คณิตศาสตร์ คือ ความสวยงาม
คณิตศาสตร์ คือ ความจริง
ติดตามชมคลิปวีดีโอได้ที่http://www.youtube.com/user/poperKM
ตอบพร้อมอ้างอิงข้อความนี้
  #8  
Old 03 สิงหาคม 2011, 17:30
Worrchet Worrchet ไม่อยู่ในระบบ
บัณฑิตฟ้า
 
วันที่สมัครสมาชิก: 15 พฤษภาคม 2011
ข้อความ: 373
Worrchet is on a distinguished road
Default

คุณ Poper ครับ ผมมีข้อสงสัยจะมาถาม

1.จากข้อ11
1.1.$\sqrt[]{a}-\sqrt[]{b}=\sqrt[]{(a+b)-2\sqrt[]{ab}},a>b$
มันคือสูตรอะไรครับแล้วเป็นของม.อะไร
1.2.แล้วตัวไหนคือ$\sqrt[]{a},$\sqrt[]{b}

2. x^{2}+y^{2}=16
x+y=4 ได้ไหมครับ?

3.$(a-b)(a^{2}+ab+b^{2})=16$
$(a-b)(a^{2}-2ab+b^{2}+3ab)=16$
$(a-b)[(a-b)^{2}+3\cdot5]=16$<---ผมงงว่า $3ab$ หายไปไหน

4.จากข้อ13
$A=\sqrt[]{9+\sqrt[]{77}}$
คูณ$\sqrt[]{2}$เข้าไปใช่ไหมครับ
$;\sqrt[]{2}A=\sqrt[]{18+2\sqrt[]{77}}$
$A=\sqrt[]{18+2\sqrt[]{77}}
$\frac{\sqrt[]{18+2\sqrt[]{77}}}{\sqrt[]{2}}$
$=\frac{\sqrt[]{11}+\sqrt[]{7}}{\sqrt[]{2}}$<---ทำอย่างไรจึงออกมาเป็นแบบนี้ครับ

5.จากข้อ14
พอแทนค่ากลับ
$\sqrt[]{(a^{2}+9a)^{2}+18(a^{2}+9a)+81}-2005^2$
$\sqrt[]{(a^2+9a+9)^{2}}-(a+3)^{2}$<---$\sqrt[]{(a^2+9a+9)^{2}}$เกิดจากอะไรครับ

6.จากข้อ15
6.1. 1000000000000
6.2. +9 มาจากไหนครับ

7.กำหนดให้ n! = n(n-1)(n-2)...3•2•1 เมื่อให้ n เป็นจน.เต็มบวก $(1!+2!+3!+...+2547!)^{(2!+3!+4!+...+2547!)^{(3!+4!+5!+...+2547!)^{...(2547)!}}}$ = A•10^n& เมื่อ $2 \leq A <10$ และ n เป็นจน.เต็มของทศนิยมหลักสุดท้ายที่ไม่ใช่ 0 คือเลขใด (โครงการสรรหานร.ที่มีความสามารถพิเศษทางคณิตศาสตร์)
เนื่องจาก 1!+2!+3!+...+2547! จะมีเลขลงท้ายด้วย 3
กำหนด $x = (1!+2!+3!+...+2547!)^{(2!+3!+4!+...+2547!)^{(3!+4!+5!+...+2547!)^{...(2547)!}}}$
         $x = (เลขลงท้ายด้วย 3)^{(2!+3!+4!+...+2547!)^{(3!+4!+5!+...+2547!)^{...(2547)!}}}$
เนื่องจาก 4 หาร 4!,5!,6!,...ลงตัวเพราะมี 4 เป็นตัวประกอบ
และ 2!+3!=8 ดังนั้น 4 หารลงตัว
; (2!+3!+4!+...+2547!)^h หารด้วย 4 ลงตัว เมื่อ $h \geq 1$ และ $h$ เป็นจน.เต็ม จะเห็นว่า x ลงท้ายด้วย 1
เมื่อจัดรูป x ให้อยู่ในรูป A•10^n  จะได้ทศนิยมหลักสุดท้ายของ A คือ 1

7.1.ช่วยขยายความตรง $4k$ อีกนิดนะครับว่า $k$ มันคืออะไรแล้วคูณด้วย4ทำไมขอบคุณครับ

7.2.ที่ใช้เลข4หารเลขยกกำลังเพราะว่า4เกิดจากการหาเลขลงท้ายของ3ใช่ไหมครับ
เพราะว่าสามเป็นเลขลงท้าย ซึ่งการหาเลขลงท้ายเกิดจากนำสามไปยกกำลัง
ผลออกมาคือเลขลงท้ายมันซำ้กันทุกๆ4ตัวจึงใช้4หารเลขยกกำลัง
จากนั้นเมื่อนำ4ไปหารปรากฏว่าลงตัวเลขลงท้ายจึงเป็นหนึ่ง
พอเราทราบว่าเลขลงท้ายคือ1เราก็จัดรูปเข้า"สัญญากรวิทยาศาสตร์"ตามที่โจทย์กำหนด;ทศนิยมหลักสุดท้ายก็คือ1ถูกต้องไหมครับ

8.$s=(2^{2}+2+1)+(3^{2}+3+1)+(4^{2}+4+1)+...+(10^{2}+10+1)$
$s=7+(1+2+3+...+10)+(1^{2}+2^{2}+3^{2}+...+10^{2}$<---แยกออกมาแบบนี้อย่างไร(ขอแบบละเอียดนะครับขอบคุณครับ)
$=447$

03 สิงหาคม 2011 21:09 : ข้อความนี้ถูกแก้ไขแล้ว 1 ครั้ง, ครั้งล่าสุดโดยคุณ Worrchet
ตอบพร้อมอ้างอิงข้อความนี้
  #9  
Old 04 สิงหาคม 2011, 09:45
poper's Avatar
poper poper ไม่อยู่ในระบบ
กระบี่ธรรมชาติ
 
วันที่สมัครสมาชิก: 12 พฤษภาคม 2010
ข้อความ: 2,643
poper is on a distinguished road
Send a message via MSN to poper
Default

1.1 สูตรการถอดรากซ้อนครับ น่าจะเจอใน ม. ปลายครับ
1.2 ถ้าเราจัดรูปให้ได้ตามสูตรก็จะถอดรูทได้ตามข้างขวาครับ
เช่น $\sqrt{6-2\sqrt{5}}=\sqrt{(5+1)-2\sqrt{5\cdot1}}=\sqrt{5}-\sqrt{1}=\sqrt{5}-1$
เช็คคำตอบโดยการนำมายกกำลังสอง $(\sqrt{5}-1)^2=6-2\sqrt{5}$ แสดงว่าถูกต้องครัย ตัวไหน $a,b$ ก็ดูเงื่อนไข $a>b$ ครับ
อย่างตัวอย่างนี้ $a=5,b=1$
2. ไม่ได้ครับ $x^2+y^2=(x+y)^2-2xy$ ครับ(มาจาก $x^2+2xy+y^2=(x+y)^2$ )
3. แทน $ab=5$ รึป่าวครับ มาจากข้อไหนอ่ะครับ
4. ใช่ครับคูณและหารด้วย $\sqrt{2}$ ครับ
(นอกนั้นอ่านไม่รู้เรื่องครับ)
__________________
คณิตศาสตร์ คือ ภาษาสากล
คณิตศาสตร์ คือ ความสวยงาม
คณิตศาสตร์ คือ ความจริง
ติดตามชมคลิปวีดีโอได้ที่http://www.youtube.com/user/poperKM
ตอบพร้อมอ้างอิงข้อความนี้
  #10  
Old 04 สิงหาคม 2011, 18:41
Worrchet Worrchet ไม่อยู่ในระบบ
บัณฑิตฟ้า
 
วันที่สมัครสมาชิก: 15 พฤษภาคม 2011
ข้อความ: 373
Worrchet is on a distinguished road
Default

มันก็ชัดนะครับ ตั้งแต่ข้อ5ไป ผมมีโจทย์มาถามอีกแล้วนะครับ
ตอบพร้อมอ้างอิงข้อความนี้
  #11  
Old 04 สิงหาคม 2011, 21:41
nongtum's Avatar
nongtum nongtum ไม่อยู่ในระบบ
ผู้พิทักษ์กฎทั่วไป
 
วันที่สมัครสมาชิก: 10 เมษายน 2005
ข้อความ: 3,246
nongtum is on a distinguished road
Default

เข้ามาช่วยแกะลายแทง น่าจะทำให้อ่านรู้เรื่องขึ้นนะครับ

ปล. 1. ตอนพิมพ์สัญลักษณ์ปนกับข้อความ หรือระหว่างข้อความ เว้นวรรคนิดนึงนะครับ (ย่อหน้า ปัดบรรทัด) จะได้อ่านง่ายขึ้น
2. ก่อนจะยืนยันว่าชัด ลองอ่านเอง แก้ไขที่ผิดเองดูก่อนสักรอบ(หรือหลายๆรอบ)ก่อนดีไหมครับ

อ้างอิง:
ข้อความเดิมเขียนโดยคุณ Worrchet View Post
คุณ Poper ครับ ผมมีข้อสงสัยจะมาถาม

1.จากข้อ11
1.1.$\sqrt[]{a}-\sqrt[]{b}=\sqrt[]{(a+b)-2\sqrt[]{ab}},a>b$
มันคือสูตรอะไรครับ แล้วเป็นของม.อะไร
1.2.แล้วตัวไหนคือ$\sqrt[]{a}, \sqrt[]{b}$

2. $x^{2}+y^{2}=16$
$x+y=4$ ได้ไหมครับ?

3.$(a-b)(a^{2}+ab+b^{2})=16$
$(a-b)(a^{2}-2ab+b^{2}+3ab)=16$
$(a-b)[(a-b)^{2}+3\cdot5]=16$ <---ผมงงว่า $3ab$ หายไปไหน

4.จากข้อ13
$A=\sqrt[]{9+\sqrt[]{77}}$
คูณ$\sqrt[]{2}$ เข้าไปใช่ไหมครับ
$;\sqrt[]{2}A=\sqrt[]{18+2\sqrt[]{77}}$
$A=\dfrac{\sqrt[]{18+2\sqrt[]{77}}}{\sqrt[]{2}}$
$=\dfrac{\sqrt[]{11}+\sqrt[]{7}}{\sqrt[]{2}}$<---ทำอย่างไรจึงออกมาเป็นแบบนี้ครับ

5.จากข้อ 14
พอแทนค่ากลับ
$\sqrt[]{(a^{2}+9a)^{2}+18(a^{2}+9a)+81}-2005^2$
$\sqrt[]{(a^2+9a+9)^{2}}-(a+3)^{2}$ <--- $\sqrt[]{(a^2+9a+9)^{2}}$ เกิดจากอะไรครับ

6.จากข้อ 15
6.1. 1000000000000
6.2. +9 มาจากไหนครับ

7.กำหนดให้ $n! = n(n-1)(n-2)...3\cdot 2\cdot1$ เมื่อให้ $n$ เป็นจน.เต็มบวก
$$(1!+2!+3!+...+2547!)^{(2!+3!+4!+...+2547!)^{(3!+4!+5!+...+2547!)^{...(2547)!}}} = A\cdot 10^n$$ เมื่อ $2 \leq A <10$ และ $n$ เป็นจน.เต็มของทศนิยมหลักสุดท้ายที่ไม่ใช่ 0 คือเลขใด
(โครงการสรรหานร.ที่มีความสามารถพิเศษทางคณิตศาสตร์)

เนื่องจาก 1!+2!+3!+...+2547! จะมีเลขลงท้ายด้วย 3
กำหนด $x = (1!+2!+3!+...+2547!)^{(2!+3!+4!+...+2547!)^{(3!+4!+5!+...+2547!)^{...(2547)!}}}$
         $$x = (เลขลงท้ายด้วย 3)^{(2!+3!+4!+...+2547!)^{(3!+4!+5!+...+2547!)^{...(2547)!}}}$$
เนื่องจาก 4 หาร 4!,5!,6!,...ลงตัวเพราะมี 4 เป็นตัวประกอบ และ 2!+3!=8 ดังนั้น 4 หารลงตัว
$(2!+3!+4!+...+2547!)^h$ หารด้วย 4 ลงตัว เมื่อ $h \geq 1$ และ $h$ เป็นจน.เต็ม จะเห็นว่า $x$ ลงท้ายด้วย 1
เมื่อจัดรูป $x$ ให้อยู่ในรูป $A\cdot 10^n$  จะได้ทศนิยมหลักสุดท้ายของ $A$ คือ 1

7.1.ช่วยขยายความตรง $4k$ อีกนิดนะครับว่า $k$ มันคืออะไรแล้วคูณด้วย4ทำไมขอบคุณครับ

7.2.ที่ใช้เลข 4 หารเลขยกกำลังเพราะว่า 4 เกิดจากการหาเลขลงท้ายของ3ใช่ไหมครับ
เพราะว่าสามเป็นเลขลงท้าย ซึ่งการหาเลขลงท้ายเกิดจากนำสามไปยกกำลัง
ผลออกมาคือเลขลงท้ายมันซ้ำกันทุกๆ 4 ตัว จึงใช้ 4 หารเลขยกกำลัง
จากนั้นเมื่อนำ 4 ไปหารปรากฏว่าลงตัวเลขลงท้ายจึงเป็นหนึ่ง
พอเราทราบว่าเลขลงท้ายคือ 1 เราก็จัดรูปเข้า "สัญญากรวิทยาศาสตร์" ตามที่โจทย์กำหนด;
ทศนิยมหลักสุดท้ายก็คือ 1 ถูกต้องไหมครับ

8.$s=(2^{2}+2+1)+(3^{2}+3+1)+(4^{2}+4+1)+...+(10^{2}+10+1)$
$s=7+(1+2+3+...+10)+(1^{2}+2^{2}+3^{2}+...+10^{2})$ <---แยกออกมาแบบนี้อย่างไร(ขอแบบละเอียดนะครับขอบคุณครับ)
$=447$
__________________
คนไทยร่วมใจอย่าใช้ภาษาวิบัติ
ฝึกพิมพ์สัญลักษณ์สักนิด ชีวิต(คนตอบและคนถาม)จะง่ายขึ้นเยอะ (จริงๆนะ)

Stay Hungry. Stay Foolish.

04 สิงหาคม 2011 21:43 : ข้อความนี้ถูกแก้ไขแล้ว 1 ครั้ง, ครั้งล่าสุดโดยคุณ nongtum
ตอบพร้อมอ้างอิงข้อความนี้
  #12  
Old 05 สิงหาคม 2011, 10:02
Worrchet Worrchet ไม่อยู่ในระบบ
บัณฑิตฟ้า
 
วันที่สมัครสมาชิก: 15 พฤษภาคม 2011
ข้อความ: 373
Worrchet is on a distinguished road
Default

ขอบคุณ คุณNongtum มากๆครับ
ผมขออนุญาตลบโจทย์เก่าๆทิ้งไปนะครับ เพราะมีแต่ลายแทง ขอบคุณทุกคนโดยเฉพาะคุณ Poperครับ

05 สิงหาคม 2011 10:15 : ข้อความนี้ถูกแก้ไขแล้ว 3 ครั้ง, ครั้งล่าสุดโดยคุณ Worrchet
ตอบพร้อมอ้างอิงข้อความนี้
  #13  
Old 05 สิงหาคม 2011, 12:31
Worrchet Worrchet ไม่อยู่ในระบบ
บัณฑิตฟ้า
 
วันที่สมัครสมาชิก: 15 พฤษภาคม 2011
ข้อความ: 373
Worrchet is on a distinguished road
Default

ช่วยเชคให้หน่อยได้ไหมครับว่าทำไมมัน กลายเป็นลายแทงอะครับ

1.12.ผลบวกของคำตอบทั้งหมดของสมการ         $(x^{2}-3x-4)^{3}+(2x^{2}+x-1)^{3}=(3x^{2}-2x-5)^{3}$ มีค่าเป็นเท่าใด (โอลิมปิก)

กำหนดให้ $a=x^{2}-3x-4=2x^{2}+x-1$
แสดงว่า $a+b =3x^{2}-2x-5$
โจทย์ 
$(x^{2}-3x-4)^{3}+(2x^{2}+x-1)^{3}$
=$(3x^{2}-2x-5)^{3}$
$;a^{3}+b^{3}$
=$(a+b)^{3}$
$a^{3}+b^{3}$
=$a^{3}+b^{3}+3a^{2}b+3ab^{2}$
$3a^{2}b+3ab^{2}$
$3ab(a+b)=0$
แสดงว่า $a=0,b=0,a+b=0$ <---แทนค่าที่สมการไหนครับถ้าแทนใน $3ab(a+b)=0$ ไม่ว่าจะแทนตนรงไหนก็มีค่าเป็น0ไม่ใช่หรอครับ.
ถ้า $a=0$ ; $(x^{2}-3x-4)=0$ $;x=4,-1$ 
ถ้า $b=0$ ; $(2x^{2}+x-1)=0$ ;$x=\frac{1}{2},-1$ 
ถ้า $a+b=0$ ; $(3x^{2}-2x-5)=0$ $;x=\frac{5}{3},-1$
แสดงว่าคำตอบของสมการคือ $-1$,$\frac{1}{2}$,$\frac{5}{3}$,$4$
ดังนั้น ผลบวกของคำตอบ$=5\frac{1}{6}$


2.กำหนด$A=(1!)^{2000}+2(2!)^{2000}+3(3!)^{2000}+...+2000(2000!)^{2000}$ จงหาเศษเหลือจากการหารAด้วย7(มหิดล)
เนื่องจาก $7|n!$ เมื่อ $n \geq 7$ โดยที่ n เป็นจน.เต็ม
;$7|n(n!)$
ดังนั้น เศษที่เหลือจากการหาร A ด้วย7จะมีค่าเท่ากับการหาเศษจากการหาร $(1!)^{2000}+2(2!)^{2000}+3(3!)^{2000}+...+2000(2000!)^{2000}$ ด้วย7โดยพิจารณา
$2(2!)^{2000}$
$=2^{2001}$
$=(2^3)^{667}$
$=(7+1)^{667}$
จะเห็นว่า 7หาร $2(2!)^{2000}$ จะมีเศษเหลือเท่ากับ1
ทำนองเดียวกัน $(1!)^{2000}$ หารด้วย7 เหลือเศษ1
$3(3!)^{2000}$ หารด้วย7 เหลือเศษ3
$4(4!)^{2000}$ หารด้วย7 เหลือเศษ1
$5(5!)^{2000}$ หารด้วย7 เหลือเศษ5
$6(6!)^{2000}$ หารด้วย7 เหลือเศษ6<---ทำไมคิดถึงแค่ $6!$ อะครับถ้าเราคิดมากกว่านี้ผลบวกก็จะมากกว่านี้ไม่ใช่หรอครับ
ดังนั้น $(1!)^{2000}+2(2!)^{2000}+3(3!)^{2000}+...+2000(2000!)^{2000}$หารด้วย7จะเหลือเศษ1+1+3+1+5+6=17
แต่ 17หารด้วย 7 เหลือเศษ3
ดังนั้น Aหารด้วย 7 จะเหลือเศษ3

ขอบคุณครับ

edit Note: ถ้าไม่ทำตารางหรือเมตริกซ์ อย่าใช้ & ใน tex นะครับ

06 สิงหาคม 2011 15:17 : ข้อความนี้ถูกแก้ไขแล้ว 2 ครั้ง, ครั้งล่าสุดโดยคุณ Worrchet
ตอบพร้อมอ้างอิงข้อความนี้
  #14  
Old 05 สิงหาคม 2011, 19:20
Worrchet Worrchet ไม่อยู่ในระบบ
บัณฑิตฟ้า
 
วันที่สมัครสมาชิก: 15 พฤษภาคม 2011
ข้อความ: 373
Worrchet is on a distinguished road
Default

จากข้อความข้างบนที่เป็นข้อสอง ผมพิมพ์ผิดตรงไหนอะครับ พอให้แสดงผลมันก็มากองกันอะครับ

ผมมีมาถามอีกครับ
1.จงหาเศษที่ได้จากการหาร$20^{20}$ด้วย17
พิจารณา$20^{20}=(2^{4}+1)2^{16}-(2^4+1)2^{12}+(2^{4}+1)2^{8}-(2^{4}+1)2^{4}+16$<---สอนแยกหน่อย
เพราะ 17|(2^4+1)จึงจะได้ว่า เศษที่ได้จากการหาร $20^{20}$ด้วย17เท่ากับ16<---ทำไมเศษเท่ากับ16

2.จงหาค่าของ $\frac{1^4+2^4+3^4}{1^2+2^2+3^2}+\frac{1^4+3^4+4^4}{1^2+3^2+4^2}+\frac{1^4+4^4+5^4}{1^2+4^2+5^2}+...+\frac{1^4+10^4+11^4}{1^2+10 ^2+11^2}$ (สอวน.)

$S = \frac{1^4+2^4+3^4}{1^2+2^2+3^2}+\frac{1^4+3^4+4^4}{1^2+3^2+4^2}+\frac{1^4+4^4+5^4}{1^2+4^2+5^2}+...+\frac{1^4+10^4+11^4}{1^2+10^ 2+11^2}$
พิจารณา $\frac{1^4+n^4+(n+1)^2}{1^2+n^2+(n+1)^2}$ <---มันสมมติตัวแปรมาจากก้อนไหนครับ
$= \frac{1^4+n^4+n^4+4n^3+6n^2+4n+1}{1+n^2+n^2+2n+1}$<---อยากทราบว่า $n^4+n^4+4n^3+6n^2+4n+1$มาจากไหนครับ
$= \frac{n^4+2n^3+3n^2+2n+1}{n^2+n+1}$
.
.
.
EDIT: มี $ เกินมาหนึ่งตัวครับ

05 สิงหาคม 2011 20:10 : ข้อความนี้ถูกแก้ไขแล้ว 1 ครั้ง, ครั้งล่าสุดโดยคุณ nongtum
ตอบพร้อมอ้างอิงข้อความนี้
  #15  
Old 05 สิงหาคม 2011, 21:34
krit krit ไม่อยู่ในระบบ
จอมยุทธ์หน้าหยก
 
วันที่สมัครสมาชิก: 14 พฤษภาคม 2010
ข้อความ: 161
krit is on a distinguished road
Default

อ้างอิง:
ข้อความเดิมเขียนโดยคุณ Worrchet View Post

1.ถ้า $\frac{1}{8}=\frac{1}{2+2\sqrt[]{a+\sqrt[]{a+\sqrt[]{a+...}}}}$ และ a มีค่าเท่าใด(โอลิมปิกรอบแรก)
กำหนด $x=\frac{1}{\sqrt[]{a+\sqrt[]{a+\sqrt[]{a+...}}}}$
จากโจทย์ ; $\frac{1}{8}=\frac{1}{2+2x}$<---จากที่โจทย์กำหนดมาว่่า $x=\frac{1}{\sqrt[]{a+\sqrt[]{a+\sqrt[]{a+...}}}}$ เวลาแทนค่าใน $\frac{1}{2+2x}$ ค่ามันไม่กลายเป็น$\frac{1}{2+2(\frac{1}{\sqrt[]{a+\sqrt[]{a+\sqrt[]{a+...}}}})}$หรอครับ
?? ? ? ? ? ?$x=3$
ดังนั้น $a+\sqrt[]{a+\sqrt[]{a+\sqrt[]{a+...}}}=9$<---มาจากไหน
$a+x=9$
$a=6$
$8=2+2\sqrt{a+\sqrt{a+\sqrt{a+...}}}$
จะได้$\sqrt{a+\sqrt{a+\sqrt{a+...}}}=3,a+\sqrt{a+\sqrt{a+\sqrt{a...}}}=9$
$\therefore a=6 $
ตอบพร้อมอ้างอิงข้อความนี้
ตั้งหัวข้อใหม่ Reply


หัวข้อคล้ายคลึงกัน
หัวข้อ ผู้ตั้งหัวข้อ ห้อง คำตอบ ข้อความล่าสุด
มาดาวน์โหลดโปรแกรม LaTeX กัน mercedesbenz ซอฟต์แวร์คณิตศาสตร์ 13 13 สิงหาคม 2009 12:40
รู้จักคำสั่ง LaTeX เบื้องต้น TOP ปัญหาคณิตศาสตร์ทั่วไป 7 13 สิงหาคม 2009 12:35
ใช้Latexไม่ได้ กรza_ba_yo ปัญหาการใช้เว็บบอร์ด 2 26 กันยายน 2008 17:08
เครื่องคอมฯไม่อ่าน latex bell18 ปัญหาการใช้เว็บบอร์ด 4 13 ธันวาคม 2007 23:10


กฎการส่งข้อความ
คุณ ไม่สามารถ ตั้งหัวข้อใหม่ได้
คุณ ไม่สามารถ ตอบหัวข้อได้
คุณ ไม่สามารถ แนบไฟล์และเอกสารได้
คุณ ไม่สามารถ แก้ไขข้อความของคุณเองได้

vB code is On
Smilies are On
[IMG] code is On
HTML code is Off
ทางลัดสู่ห้อง


เวลาที่แสดงทั้งหมด เป็นเวลาที่ประเทศไทย (GMT +7) ขณะนี้เป็นเวลา 13:16


Powered by vBulletin® Copyright ©2000 - 2024, Jelsoft Enterprises Ltd.
Modified by Jetsada Karnpracha